A 69.0-kg astronaut is floating in space, luckily he has his trusty 28.0-kg physics book. He throws his physics book and accelerates at 0.0130 m/s2 in the opposite direction. What is the magnitude of the acceleration of the physics book?

Answers

Answer 1

Answer:

0.032 [tex]m/s^2[/tex]

Explanation:

Given :

Weight of the astronaut = 69 kg

Weight of the physics book = 28 kg

Acceleration of the astronaut = 0.0130 [tex]m/s^2[/tex]

The force that is applied on the astronaut :

[tex]F=ma[/tex]

   [tex]$=69 \times 0.013$[/tex]

   = 0.897 N

Therefore, by Newton's 3rd law, we know that the force applied on the physics book is also F = 0.897 N

Therefore, the acceleration of the physics book is given by :

[tex]$a = \frac{\text{Force on physics book}}{\text{mass of physics book}}$[/tex]

[tex]$a = \frac{0.897}{28}$[/tex]

a = 0.032 [tex]m/s^2[/tex]

Hence, the acceleration of the physics book is  0.032 [tex]m/s^2[/tex].

Answer 2

Answer:

The acceleration of astronaut is 5.27 x 10^-3 m/s^2.

Explanation:

mass of astronaut, M = 69 kg

Mass of book, m = 28 kg

acceleration of book, a = 0.013 m/s^2

Let the acceleration of astronaut is A.

According to the Newton's third law, for every action there is an equal and opposite reaction.

So, the force acting on the book is same as the force acting on the astronaut but the direction is opposite to each other.

M A = m a

69 x A = 28 x 0.013

A = 5.27 x 10^-3 m/s^2


Related Questions

Outline five ways of varying the force on a current-carrying conductor in a magnetic field. (7 marks) ​

Answers

Is this it? I think it is :)

1. A flywheel begins rotating from rest, with an angular acceleration of 0.40 rad/s. a) What will its angular velocity be 3 seconds later? b) What angle will it have turned through in that time? ​

Answers

Answer:

(a) 1.2 rad/s

(b) 1.8 rad

Explanation:

Applying,

(a) α = (ω-ω')/t................ Equation 1

Where α = angular acceleration, ω = final angular velocity, ω' = initial angular velocity, t = time.

From the question,

Given: α = 0.40 rad/s², t = 3 seconds, ω' = 0 rad/s (from rest)

Substitute these values into equation 1

0.40 = (ω-0)/3

ω = 0.4×3

ω = 1.2 rad/s

(b) Using,

∅ = ω't+αt²/2.................. Equation 2

Where ∅ = angle turned.

Substitutting the values above into equation 2

∅ = (0×3)+(0.4×3²)/2

∅ = 1.8 rad.

I NEED YOUR HELP!!
Calculate the minimum area moment of inertia for a rectangular cross-section with side lengths 6 cm and 4 cm.


52 cm4


72 cm4


32 cm4


24 cm4


2 cm4

Answers

Answer:

Minimum area of rectangle = 24 centimeter²

Explanation:

Given:

Length of rectangle = 6 centimeter

Width of rectangle = 4 centimeter

Find:

Minimum area of rectangle

Computation :

Minimum area of rectangle = Length of rectangle x Width of rectangle

Minimum area of rectangle = 6 x 4

Minimum area of rectangle = 24 centimeter²

One ball is aprojected in the uapward directon with a certain velocity ‘v’ and other

is thrown downwards with the same velocity ​

Answers

Complete question is;

One ball is projected in the upward direction with a certain velocity ‘v’ and other is thrown downwards with the same velocity at an angle θ.

The ratio of their potential energies at highest points of their journey, will be:

Answer:

u² : (u cos θ)²

Explanation:

Maximum potential energy for the first ball will be at a maximum height of;

H = u²/2g

Thus;

PE = mg(u²/2g)

For second ball at an angle θ, maximum PE will occur at a max height of (u cos θ)²/2g

PE = mg((u cos θ)²/2g)

The ratios of the potential energies are;

mg(u²/2g) : mg((u cos θ)²/2g)

mg will will cancel out since they are of same mass.

Thus;

(u²/2g) : (u cos θ)²/2g

Again 2g will cancel out to give;

u² : (u cos θ)²

A 34-m length of wire is stretched horizontally between two vertical posts. The wire carries a current of 68 A and experiences a magnetic force of 0.16 N. Find the magnitude of the earth's magnetic field at the location of the wire, assuming the field makes an angle of 72.0° with respect to the wire.

Answers

Answer:

7.28×10⁻⁵ T

Explanation:

Applying,

F = BILsin∅............. Equation 1

Where F = magnetic force, B = earth's magnetic field, I = current flowing through the wire, L = Length of the wire, ∅ = angle between the field and the wire.

make B the subject of the equation

B = F/ILsin∅.................. Equation 2

From the question,

Given: F = 0.16 N, I = 68 A, L = 34 m, ∅ = 72°

Substitute these values into equation 2

B = 0.16/(68×34×sin72°)

B = 0.16/(68×34×0.95)

B = 0.16/2196.4

B = 7.28×10⁻⁵ T

distance of distinct vision.
is placed at a distance less than the distance of near point, its image o
will be blurred. Hence human eye can not see such object clearly.
ADDITIONAL INFORMATION
distance of distinct vision for a normal eye of different age groups
Babies = 7 cm
Adults = 25 cm
erson of age 55 years and above = 100 cm
ever, in our discussion we are concerned with a normal eye of an adult so least
The foulart position of an ahiect from a human eve so that the sh​

Answers

The least distance up to which we can see the objects clearly without any strain is called least distance of distinct vision. Least distance of distinct vision for a normal human being is 25cm. For young people, the least distance of distant vision will be within 25cm which however it varies with age.

Answer:

25 you said ? thats incorecct

Explanation:

How much work does the electric field do in moving a proton from a point with a potential of 170 V to a point where it is -50 V

Answers

a) A charged particle accelerates as it moves from location A to location B. If V A = 170 V and V B = 210 V, what is the sign of the charged particle? positive negative (b) A proton gains electric potential energy as it moves from point 1 to point 2.
Physics
Two identical point charges are fixed to diagonally opposite corners of a square that is 1.5 m on a side. Each charge is +2.0 µC. How much work is done by the electric force as one of the charges moves to an empty corner? I have W(fe)= -EPE=-q[V(f)-V(i)].
physics
Two point charges are placed on the x axis. (Figure 1) The first charge, q1 = 8.00nC , is placed a distance 16.0m from the origin along the positive x axis; the second charge, q2 = 6.00nC , is placed a distance 9.00m from the origin along the negative x

Urgent please help me

Answers

1433 km

Explanation:

Let g' = the gravitational field strength at an altitude h

[tex]g' = G\dfrac{M_E}{(R_E + h)^2}[/tex]

We also know that g at the earth's surface is

[tex]g = G\dfrac{M_E}{R_E^2}[/tex]

Since g' = (2/3)g, we can write

[tex]G\dfrac{M_E}{(R_E + h)^2} = \dfrac{2}{3}\left(G\dfrac{M_E}{R_E^2} \right)[/tex]

Simplifying the above expression by cancelling out common factors, we get

[tex](R_E + h)^2 = \dfrac{3}{2} R_E^2[/tex]

Taking the square root of both sides, this becomes

[tex]R_E + h = \left(\!\sqrt{\dfrac{3}{2}}\right) R_E[/tex]

Solving for h, we get

[tex]h = \left(\!\sqrt{\dfrac{3}{2}} - 1\right) R_E= 0.225(6.371×10^2\:\text{km})[/tex]

[tex]\:\:\:\:\:= 1433\:\text{km}[/tex]

Electricity is distributed from electrical substations to neighborhoods at 13000 V. This is a 60 Hz oscillating (AC) voltage. Neighborhood transformers, seen on utility poles, step this voltage down to the 120 V that is delivered to your house.

Required:
a. How many turns does the primary coil on the transformer have if the secondary coil has 130 turns?
b. No energy is lost in an ideal transformer, so the output power Pout from the secondary coil equals the input power Pin to the primary coil. Suppose a neighborhood transformer delivers 280 A at 120 V. What is the current in the 1.3×10^4 V line from the substation?

Answers

Answer:

a)  N₁ = 14083 turns,  b)   I₁ = 2.58 A

Explanation:

The relationship that describes the relationship between the primary and secondary of the transformer is

         [tex]\frac{V_2}{N_2} = \frac{V_1}{N_1}[/tex]

a) They indicate that the secondary has N2 = 130 turns, the turns of the primary are

         N₁ = [tex]N_2 \frac{V_1}{V_2}[/tex]

         N₁ = [tex]130 \ \frac{13000}{120}[/tex]

         N₁ = 14083 turns

b) since there are no losses, the power of the neighboring transformer is

          P = V I

          P = 120 280

          P = 33600 W

this is the same power of the substation

          P = V₁  I₁

          I₁ = P / V₁

          I₁ = 33600/13000

          I₁ = 2.58 A

What is the meant of by renewable energy and non-renewrable with example of each.​

Answers

Answer:

Renewable energy is a type of energy that can be renewed easily, such as sunlight. By using Solar panels to collect the suns energy, we are not depleting it, so this source is renewable.

Non-renewable  energy is something that cannot easily be replenished. An example would be oil because oil takes millions of years to form and cannot be renewed easily.

Wind instruments like trumpets and saxophones work on the same principle as the "tube closed on one end" that we examined in our last experiment. What effect would it have on the pitch of a saxophone if you take it from inside your house (at 76 degrees F) to the outside on a cold day when the outside temperature is 45 degrees F ?

Answers

Answer:

The correct answer would be - Low pitch.

Explanation:

As it is known that if frequency increases then pitch will be increase as well as pitch depends on frequency, Now for the question it is mentioned that the tube closed on one end frequency is:

f = v/2l

Where,

l = length of the tube

v = velocity of longitudinal wave of gas filled in the tube

Now increase with the temperature the density of the gas decreases and velocity v is inversely proportional to density of gas so velocity increases. So if there is an increase in frequency so pitch also increases. As the temperature inside the house is at 750 F more than outsideat 450 Fso pitch is more inside and the pitch is low outside.

A car travelling at 14.0 m/s approaches a traffic light. The driver applies the brakes and is able to come to halt in 5.6 s. Determine the average acceleration of the car during this time interval.

Answers

Answer:

[tex]a=2.5\ m/s^2[/tex]

Explanation:

Given that,

Initial speed of the car, u = 14 m/s

Finally, it comes to rest, v = 0

Time, t = 5.6 s

We need to find the average acceleration of the car during this time interval. We know that,

[tex]a=\dfrac{v-u}{t}\\\\a=\dfrac{0-14}{5.6}\\\\a=-2.5\ m/s^2[/tex]

So, the acceleration of the car is [tex]2.5\ m/s^2[/tex] in the opposite direction of motion.

A 1500kg car start from rest and increases it velocity to 30mls in a time of 25sec. calculate the distance the car travel, how much force was use, how much work was done.​

Answers

Answer:

workdone= 1/2mv^2

1/2×1500×30^2

675000J

In a similar rolling race (no slipping), the two objects are a solid cylinder and hollow cylinder of the same radius and mass. Which reaches the bottom first

Answers

Answer:

solid cylinder

Explanation:

the object that arrives first is the object that has more speed, let's use the concepts of energy

starting point. Highest point

         Em₀ = U = m g h

final point. Lowest point

         Em_f = K = ½ mv² + ½ I w²

since the body has rotational and translational movement

how energy is conserved

         m g h = ½ mv² + ½ I w²

linear and angular velocity are related

          v = w r

          w = v / r

we substitute

          m g h = ½ mv² + ½ I (v/r) ²

          mg h = ½ v² (m + I /r²)

          v = [tex]\sqrt{2gh \ \frac{m}{m + \frac{I}{r^2} } }[/tex]

           

the tabulated moments of inertia for the bodies are

solid cylinder     I = ½ m r²

hollow cylinder  I = m r²

we look for the speed for each body

solid cylinder

          v₁ = [tex]\sqrt {2gh} \ \sqrt{\frac{m}{m + m/2} }[/tex]

          v₁ = [tex]\sqrt{2gh} \ \sqrt{2/3}[/tex]

let's call    v₀ = [tex]\sqrt{2gh}[/tex]

         v₁ = 0.816 v₀

hollow cylinder

          v₂ = [tex]\sqrt{2gh } \ \sqrt{\frac{m}{m+ m} }[/tex]

          v₂ = v₀ √½

          v₂ = 0.707 v₀

Therefore, the body that has the highest speed is the solid cylinder and since time is the inverse of speed, this is the body that spends less time to reach the bottom, that is, it is the first to arrive

derive expression for pressure exerted by gas ​

Answers

From kinetic theory of gases, the pressure exerted by a gas is given by velocity of gas molecules. m = Mass of each molecule of a gas. But by assumptions of the kinetic theory of gases the average kinetic energy of a molecule is constant at a constant temperature.

A car driving down a road runs of gas and will eventually stop because of:
A. Friction
B. Thrust
C. It will remain in motion forever
OD. Gravity

Answers

The answer is A. Friction
The answer is Friction

A wire carrying a 23.0 A current passes between the poles of a strong magnet such that the wire is perpendicular to the magnet's field, and there is a 2.45 N force on the 3.00 cm of wire in the field. What is the average field strength (in T) between the poles of the magnet?

Answers

Answer:

3.55 T

Explanation:

Applying,

F = BILsin∅.............. Equation 1

Where F = Force, B = magnetic Field, I = current, L = Length of the wire, ∅ = Angle between the wire and the magnetic field

make B the subject of the equation

B = F/ILsin∅.................. Equation 2

From the question,

Given: F = 2.45 N, L = 3.00 cm = 0.03 m, I = 23.0 A, ∅ = 90° (Perpendicular)

Substitute these values into equation 2

B = 2.45/(0.03×23×sin90)

B = 2.45/0.69

B = 3.55 T

what can you do to keep you BMI under weight

Answers

Here are some healthy ways to gain weight when you're underweight:

Eat more frequently. When you're underweight, you may feel full faster. Eat five to six smaller meals during the day rather than two or three large meals.

Choose nutrient-rich foods. As part of an overall healthy diet, choose whole-grain breads, pastas and cereals; fruits and vegetables; dairy products; lean protein sources; and nuts and seeds.

Try smoothies and shakes. Don't fill up on diet soda, coffee and other drinks with few calories and little nutritional value. Instead, drink smoothies or healthy shakes made with milk and fresh or frozen fruit, and sprinkle in some ground flaxseed. In some cases, a liquid meal replacement may be recommended.

Watch when you drink. Some people find that drinking fluids before meals blunts their appetite. In that case, it may be better to sip higher calorie beverages along with a meal or snack. For others, drinking 30 minutes after a meal, not with it, may work.

Make every bite count. Snack on nuts, peanut butter, cheese, dried fruits and avocados. Have a bedtime snack, such as a peanut butter and jelly sandwich, or a wrap sandwich with avocado, sliced vegetables, and lean meat or cheese.

Top it off. Add extras to your dishes for more calories — such as cheese in casseroles and scrambled eggs, and fat-free dried milk in soups and stews.

Have an occasional treat. Even when you're underweight, be mindful of excess sugar and fat. An occasional slice of pie with ice cream is OK. But most treats should be healthy and provide nutrients in addition to calories. Bran muffins, yogurt and granola bars are good choices.

Exercise. Exercise, especially strength training, can help you gain weight by building up your muscles. Exercise may also stimulate your appetite.

Eat more frequently. When you're underweight, you may feel full faster. ...
Choose nutrient-rich foods. ...
Try smoothies and shakes. ...
Watch when you drink. ...
Make every bite count. ...
Top it off. ...
Have an occasional treat. ...
Exercise.

If a pendulum's length is 2.00 m and ag = 9.80 m/s, how many complete oscillations does the pendulum make in 5.00 min?​

Answers

Answer:

Number of oscillation = 106 oscillations

Explanation:

Given the following data;

Length = 2 mAcceleration due to gravity, g = 9.8 m/s²Time = 5 minutes

To find how many complete oscillations the pendulum makes in 5.00 min;

First of all, we would determine the period of oscillation of the pendulum using the following formula;

[tex] T = 2 \pi \sqrt{\frac{l}{g}} [/tex]

Where;

T is the period.l is the length of the pendulum.g is acceleration due to gravity.

Substituting into the formula, we have;

[tex] T = 2 * 3.142 \sqrt{\frac{2}{9.8}} [/tex]

[tex] T = 6.284 \sqrt{0.2041} [/tex]

[tex] T = 6.284 * 0.4518 [/tex]

Period, T = 2.84 seconds

Next, we would determine the number of complete oscillation in 5 minutes;

We would have to convert the time in minutes to seconds.

Conversion:

1 minutes = 60 seconds

5 minutes = X seconds

Cross-multiplying, we have;

X = 5 * 60 = 300 seconds

Mathematically, the number of oscillation of a pendulum is given by the formula;

[tex] Number \; of \; oscillation = \frac {Time}{Period} [/tex]

Substituting into the formula, we have;

[tex] Number \; of \; oscillation = \frac {300}{2.84} [/tex]

Number of oscillation = 105.63 ≈ 106 oscillations

Number of oscillation = 106 oscillations

Calculate the buoyant force due to the surrounding air on a man weighing 600 N . Assume his average density is the same as that of water. Suppose that the density of air is 1.20 kg/m3.

Answers

Answer:

[tex]F_b= 0.720 N[/tex]

Explanation:

From the question we are told that:

Weight [tex]W=600N[/tex]

Average density [tex]\rho=1.20kg/m^3[/tex]

Mass

[tex]m=\frac{W}{g}[/tex]

[tex]m=\frac{600}{9.81}[/tex]

[tex]m=61.22kg[/tex]

Generally the equation for Volume is mathematically given by

[tex]V =\frac{ mass}{density}[/tex]

[tex]V= \frac{61.22}{1000}[/tex]

[tex]V=0.06122 m^3[/tex]

Therefore

Buoyant force [tex]F_b[/tex]

[tex]F_b=\rho*V*g[/tex]

[tex]F_b= rho_air*V*g[/tex]

[tex]F_b= 0.720 N[/tex]

A pitching machine is programmed to pitch baseballs horizontally at a speed of 126 km/h. The machine is mounted on a truck and aimed forward. As the truck drives toward you at a speed of 85 km/h, the machine shoots a ball toward you. For each of the object pairings listed below, determine the correct relative speed.

a. The speed of the pitching machine relative to the truck
b. The speed of the pitched bell relative to the truck
c. The speed of the pitching machine relative to you
d. The speed of the pitched ball relative to you

Answers

Explanation:

a) zero, since the machine is mounted on the truck

b) 126 km/hr

c) 85 km/hr

d) 126 km/hr + 85 km/hr = 211 km/hr

An alternating voltage is connected in series with a resistance R and an inductance L If the potential drop across the
resistance is 200 V and across the inductance is 100V
then the applied voltage is
V 223.6
V 2006
V 300
V50
Please help me

Answers

Answer:

oh my God I got really confused right now

1) Consider an electric power transmission line that carries a constant electric current of i = 500 A. The cylindrical copper cable used to transmit this current has a diameter o = 2.00 cm and a length L = 150 km. If there are 8.43x10^28 free electrons per cubic meter (m^3 ) in the cable, calculate how long it would take for an electron to cross the entire length of the transmitter line.

Answers

Answer:  

t = 1.27 x 10⁹ s  

Explanation:  

First, we will find the volume of the wire:

Volume = V = AL  

where,  

A = Cross-sectional area of wire = πr² = π(1 cm)² = π(0.01 m)² = 3.14 x 10⁻⁴ m²  

L = Length of wire = 150 km = 150000 m  

Therefore,    

V = 47.12 m³

 

Now, we will find the number of electrons in the wire:  

No. of electrons = n = (Electrons per unit Volume)(V)  

n = (8.43 x 10²⁸ electrons/m³)(47.12 m³)  

n = 3.97 x 10³⁰ electrons  

Now, we will use the formula of current to find out the time taken by each electron to cross the wire:

[tex]I =\frac{q}{t}[/tex]  

where,  

t = time = ?  

I = current = 500 A  

q = total charge = (n)(chareg on one electron)  

q = (3.97 x 10³⁰ electrons)(1.6 x 10⁻¹⁹ C/electron)  

q = 6.36 x 10¹¹ C  

[tex]500\ A = \frac{6.36\ x\ 10^{11}\ C}{t}\\\\t = \frac{6.36\ x\ 10^{11}\ C}{500\ A}[/tex]

Therefore,

t = 1.27 x 10⁹ s

Một ống dây điện thẳng dài có lõi sắt, tiết diện ngang của ống S = 20 cm2

, chiều dài
1 m, hệ số tự cảm L = 0,44 H. Cường độ từ trường trong ống dây là H = 0,8.103 A/m. Từ
thông gửi qua tiết diện ngang của ống bằng

3

0
1,6.10 Wb

. Cường độ dòng điện chạy

qua ống dây là

Answers

Answer:

sgsbssbduebubbeeifirjeirneejrbb8m!keoejr

d

iejejjeiie

A bird has a kinetic energy of 3 J and a potential energy of 25 J. What is the mechanical energy of the bird?

Answers

Answer:

28 j

Explanation:

because when you add you get 28

An ideal parallel plate capacitor with a cross-sectional area of 0.4 cm2 contains a dielectric with a dielectric constant of 4 and a dielectric strength of 2 x 108 V/m. The separation between the plates of the capacitor is 5 mm. What is the maximum electric charge (in nC) that can be stored in the capacitor before dielectric breakdown

Answers

Answer: [tex]283.2\times 10^{-9}\ nC[/tex]

Explanation:

Given

Cross-sectional area [tex]A=0.4\ cm^2[/tex]

Dielectric constant [tex]k=4[/tex]

Dielectric strength [tex]E=2\times 10^8\ V/m[/tex]

Distance between capacitors [tex]d=5\ mm[/tex]

Maximum charge that can be stored before dielectric breakdown is given by

[tex]\Rightarrow Q=CV\\\\\Rightarrow Q=\dfrac{k\epsilon_oA}{d}\cdot (Ed)\quad\quad [V=E\cdot d]\\\\\Rightarrow Q=k\epsilon_oAE\\\\\Rightarrow Q=4\times 8.85\times 10^{-12}\times 0.4\times 10^{-4}\times 2\times 10^8\\\\\Rightarrow Q=28.32\times 10^{-8}\\\\\Rightarrow Q=283.2\times 10^{-9}\ nC[/tex]

Answer:

The maximum charge is 7.08 x 10^-8 C.

Explanation:

Area, A = 0.4 cm^2

K = 4

Electric field, E = 2 x 10^8 V/m

separation, d = 5 mm = 0.005 m

Let the capacitance is C and the charge is q.

[tex]q = CV\\\\q=\frac{\varepsilon o A}{d}\times E d\\\\q = \varepsilon o A E\\\\q = 8.85\times 10^{-12}\times0.4\times 10^{-4}\times 2\times 10^8\\\\q = 7.08\times 10^{-8}C[/tex]

What is the name of the invisible line that runs
down the center of the axial region?

Answers

Answer:

An axis is an invisible line around which an object rotates, or spins. The points where an axis intersects with an object's surface are the object's North and South Poles.

Explanation:

The Earth's axis is represented by the red line. The white circle represents axial precission, the slow "wobble" of the axis.

Why must scientists be careful when studying
nanotechnology?

Answers

Answer:

When studying nanotechnology, scientists must be aware that their ideas may not work out. Their work could be very time consuming and cost a lot of money. Finally, scientists do not yet know all of the effects of nanotechnology on human health.

Hope it helps u:)

A stone dropped from the top of a 80m high building strikes the ground at 40 m/s after falling for 4 seconds. The stone's potential energy with respect to the ground is equal to its kinetic energy … (use g = 10 m/s 2)

A) at the moment of impact.
B) 2 seconds after the stone is released.
C) after the stone has fallen 40 m.
D) when the stone is moving at 20 m/s.

At the moment of impact both Kinetic Energy and Potential Energy should be 0, right? So it can't be A), right? Or is this wrong? Is it indeed A)? Please show work and explain it well.

Answers

Answer:

Explanation:

The answer is C because the building is 80 meters high. Before the stone is dropped, it has ONLY potential energy since kinetic energy involves velocity and a still stone has no velocity. At impact, there is no potential energy because potential energy involves the height of the stone relative to the ground and a stone ON the ground has no height; here there is ONLY kinetic.

From the First Law of Thermodynamics, we know that energy cannot be created or destroyed, it can only change form. Therefore, that means that at the halfway point of 40 meters, half of the stone's potential energy has been lost, and it has been lost to kinetic energy. Here, at 40 meters, there is an equality between PE and KE. It only last for however long the stone is AT 40 meters, which is probably a millisecond of time, but that's where they are equal.

find the exit angle relative to the horizontal in an isosceles triangle with 36 °​

Answers

what

what

what

what

sorry

sorrry

sorry

Other Questions
What is the axis of symmetry for the graph of y 4x = 7 x2 ? hdjehhwhehbwbwnnwnwnjejdiidjsnnwnnwnrnfjfjjdjejnwnnrnfnfjjjw Determine which of the following statements is true about energy in ecosystems. A. Producers are able to convert 40% of the suns energy into chemical energy. B. 30% of the suns energy cannot be absorbed by producers because it is in the wrong wavelength of light. C. Producers only absorb 40% of the suns energy, most of which is lost during photosynthesis. D. Producers absorb 5% of the suns energy, and reflect 5% of the suns energy. Trucker's Heaven Diner uses hundreds of plates, glasses, and cups each day. The dishwashers start a new load of dishes every 1 1/4 hours. If the dishwashers work for 10 hours . How many loads off dishes do they wash in that time?A: 1/2B: 8C: 8 3/4D: 12 1/3 Question 10 of 10 Why was it unusual that African Americans supported FDR? O A. FDR was white, and most African Americans only voted for black candidates. O B. FDR was a Democrat, and most African Americans voted Republican. O C. FDR was from the North, and most African Americans voted only for Southerners. O D. FDR was a Democrat, but southern Democrats opposed him. Del texto Pedro Paramo, extrae:1. Una oracin interrogativa2. Una oracin exclamativa3. Una oracin imperativa4. Una oracin enunciativa afirmativa5. Una oracin enunciativa negativa g An Hfr strain is produced when Select one: A. an F episome is transferred to a new cell. B. an F episome leaves the E. coli chromosome. C. an F episome leaves the host chromosome and removes some of the host chromosome. D. an F episome integrates into the E. coli chromosom 2 What are the major challenges to tackle your district? Consider the reaction: CaCO3(s)CaO(s) CO2(g) Using standard absolute entropies at 298K, calculate the entropy change for the system when 1.58 moles of CaCO3(s) react at standard conditions. A shunt regulator utilizing a zener diode with an incremental resistance of 8 ohm is fed through an 82-Ohm resistor. If the raw supply changes by 1.0 V, what is the correspondig change in the regulated outut voltage? What is the difference between the temperature - 7 Celsius and - 12 Celsius on a scatter diagram Which expression is equivalent to the following complex fraction? Please, Help me :'u - CORRECT THE MISTAKES.What Time Is?It The Same PlaceShe Usually Go To The Church To get maximum current in a circuit, the resistance should be in _____1)series2)parallel How did the sailors help the colonists? Which phrase best describes the resolution in a typical flash fiction story?disappointing and confusingfull of emotioncarefully explainedsomewhat ambiguous A pooled variance is an estimated weighted variance made up of several different populations when the mean of each population may be different, but one may assume that the variance of each population is the same.a. Trueb. False A suspension of yeast cells is being grown under anaerobic conditions such that glucose is degraded to ethanol and carbon dioxide. To follow this process by monitoring the release of CO214, at which positions in the glucose molecule would the C14 label need to be incorporated Nobody needs to go anywhere else. We are all, if we only knew it, already there. If I only knew who in fact I am, I should cease to behave as what I think I am; and if I stopped behaving as what I think I am, I should know who I am. What in fact I am, if only the Manichee I think I am would allow me to know it, is the reconciliation of yes and no lived out in total acceptance and the blessed experience of Not-Two. In religion all words are dirty words. Anybody who gets eloquent about Buddha, or God, or Christ, ought to have his mouth washed out with carbolic soap Madison owns a retail clothing store. Her marketing consultant says that with all the technology at their disposal, customers will go back and forth across multiple channels before choosing to buy an outfit at her store. The consultant recommends that Madison consider an online-to-offline strategy to drive foot traffic to her store. What are two examples of an online-to-offline strategy